When positive integer n is divided by 13, the remainder is 2

This topic has expert replies
Moderator
Posts: 2207
Joined: Sun Oct 15, 2017 1:50 pm
Followed by:6 members

Timer

00:00

Your Answer

A

B

C

D

E

Global Stats

Veritas Prep

When positive integer n is divided by 13, the remainder is 2. When n is divided by 8, the remainder is 5. How many such values are less than 180?

A. 0
B. 1
C. 2
D. 3
E. 4

OA B

User avatar
GMAT Instructor
Posts: 1449
Joined: Sat Oct 09, 2010 2:16 pm
Thanked: 59 times
Followed by:33 members

by fskilnik@GMATH » Sun Sep 30, 2018 2:24 pm
BTGmoderatorLU wrote:Veritas Prep

When positive integer n is divided by 13, the remainder is 2. When n is divided by 8, the remainder is 5. How many such values are less than 180?

A. 0
B. 1
C. 2
D. 3
E. 4
$$1 \le n \le 179\,\,\,\,\,\left( {n\,\,\,{\mathop{\rm int}} } \right)\,\,\,\,\left( * \right)$$
$$n = 13M + 2\,\,\,\,,\,\,\,M\,\,{\mathop{\rm int}} \,\,\,\,\left( {\rm{I}} \right)$$
$$n = 8J + 5\,\,\,\,,\,\,\,J\,\,{\mathop{\rm int}} \,\,\,\,\left( {{\rm{II}}} \right)$$
\[?\,\,\,:\,\,\,n\,\,\,{\text{in}}\,\,\,\left( * \right) \cap \left( {\text{I}} \right) \cap \left( {{\text{II}}} \right)\]

$$\left( * \right)\,\, \cap \,\,\left( {\rm{I}} \right)\,\,\,:\,\,\,\,\,\,1\,\, \le \,\,13M + 2\,\, \le \,\,179\,\,\,\,\,\mathop \Leftrightarrow \limits^{ - \,2} \,\,\,\,\, - 1 \le 13M \le 177\,\,\left( { = 169 + 8} \right)$$
$$ - 1 \le 13M \le 177\,\,\left( { = 169 + 8} \right)\,\,\,\,\,\mathop \Leftrightarrow \limits^{M\,\,{\mathop{\rm int}} } \,\,\,0 \le 13M \le 169\,\,\,\,\,\mathop \Leftrightarrow \limits^{:\,\,13} \,\,\,\,0 \le M \le 13$$

$$\left( * \right)\,\, \cap \,\,\left( {{\rm{II}}} \right)\,\,\,:\,\,\,\,\,\,\left\{ \matrix{
\,n - 5\,\,{\rm{divisible}}\,\,{\rm{by}}\,\,8\,\,\,\,\, \Rightarrow \,\,\,\,\,n - 5\,\,\,\,\,{\rm{even}}\,\,\,\,\, \Rightarrow \,\,\,\,\,n\,\,\,{\rm{odd}}\,\,\,\,\,\mathop \Rightarrow \limits^{\left( {\rm{I}} \right)} \,\,\,\,\,M\,\,{\rm{odd}} \hfill \cr
\,n - 5\mathop = \limits^{\left( {\rm{I}} \right)} 13M - 3\,\,{\rm{divisible}}\,\,{\rm{by}}\,\,8\,\,\,\,\, \Rightarrow \,\,\,\,\,{{13M - 3} \over 2}\,\,\,\,{\rm{divisible}}\,\,{\rm{by}}\,\,4\,\,\,\left( {***} \right)\,\,\,\,\,\, \hfill \cr} \right.$$
\[\left. \begin{gathered}
M = 1\,\,\,\,\, \Rightarrow \,\,\,\,\frac{{13 - 3}}{2} = 5\,\,\,{\text{odd}}\,\,\,\left( {***} \right)\,\,\,{\text{NO}} \hfill \\
M = 3\,\,\,\,\, \Rightarrow \,\,\,\,\frac{{13 \cdot 3 - 3}}{2} = 18\,\,\,\,\left( {***} \right)\,\,\,{\text{NO}} \hfill \\
M = 5\,\,\,\,\, \Rightarrow \,\,\,\,\frac{{13 \cdot 5 - 3}}{2} = 31\,\,{\text{odd}}\,\,\,\,\left( {***} \right)\,\,\,{\text{NO}}\,\,\,\,\,\, \hfill \\
\boxed{M = 7}\,\,\,\,\, \Rightarrow \,\,\,\,\frac{{13 \cdot 7 - 3}}{2} = 44\,\,\,\,\left( {***} \right)\,\,\,{\text{YES}} \hfill \\
M = 9\,\,\,\,\, \Rightarrow \,\,\,\,{\text{odd}}\,\,\,\,\left( {***} \right)\,\,\,{\text{NO}} \hfill \\
M = 11\,\,\,\,\, \Rightarrow \,\,\,\,\frac{{13 \cdot 11 - 3}}{2} = 70\,\,\,\,\left( {***} \right)\,\,\,{\text{NO}} \hfill \\
M = 13\,\,\,\,\, \Rightarrow \,\,\,\,{\text{odd}}\,\,\,\,\left( {***} \right)\,\,\,{\text{NO}} \hfill \\
\end{gathered} \right\}\,\,\,\,\,\,\,\,\,\, \Rightarrow \,\,\,\,\,\,\,? = 1\]

This solution follows the notations and rationale taught in the GMATH method.

Regards,
Fabio.
Fabio Skilnik :: GMATH method creator ( Math for the GMAT)
English-speakers :: https://www.gmath.net
Portuguese-speakers :: https://www.gmath.com.br

GMAT/MBA Expert

User avatar
GMAT Instructor
Posts: 16207
Joined: Mon Dec 08, 2008 6:26 pm
Location: Vancouver, BC
Thanked: 5254 times
Followed by:1268 members
GMAT Score:770

by Brent@GMATPrepNow » Sun Sep 30, 2018 2:41 pm
BTGmoderatorLU wrote:Veritas Prep

When positive integer n is divided by 13, the remainder is 2. When n is divided by 8, the remainder is 5. How many such values are less than 180?

A. 0
B. 1
C. 2
D. 3
E. 4
When it comes to remainders, we have a nice rule that says:
If N divided by D leaves remainder R, then the possible values of N are R, R+D, R+2D, R+3D,. . . etc.
For example, if k divided by 5 leaves a remainder of 1, then the possible values of k are: 1, 1+5, 1+(2)(5), 1+(3)(5), 1+(4)(5), . . . etc.

When positive integer n is divided by 13, the remainder is 2.
So, some possible values of n (less than 180) are: 2, 15, 28, 41, 54, 67, 80, 93, 106, 119, 132, 145, 158, 171

When n is divided by 8, the remainder is 5.
Now check the above list of numbers and identify all of the values that, when divided by 8, leave a remainder of 5
I've highlighted in red all that apply

Answer: B

Cheers,
Brent
Brent Hanneson - Creator of GMATPrepNow.com
Image

GMAT/MBA Expert

User avatar
GMAT Instructor
Posts: 7243
Joined: Sat Apr 25, 2015 10:56 am
Location: Los Angeles, CA
Thanked: 43 times
Followed by:29 members

by Scott@TargetTestPrep » Sun Oct 07, 2018 6:34 pm
BTGmoderatorLU wrote:Veritas Prep

When positive integer n is divided by 13, the remainder is 2. When n is divided by 8, the remainder is 5. How many such values are less than 180?

A. 0
B. 1
C. 2
D. 3
E. 4
We can create the equation:

n = 13Q + 2

So n can be 2, 15, 28, 41, 54, 67, 80, 93, ...

and

n = 8R + 5

So n can be 5, 13, 21, 29, 37, 45, 53, 61, 69, 77, 85, 93, ...

We see that the first number that satisfies both conditions is 93. To find the other numbers, we can keep adding the LCM of 13 and 8, which is 13 x 8 = 104. Therefore, the next value that satisfies both conditions is 93 + 104 = 197. However, 197 is already greater than 180, so we only have one value, namely 93, that is less than 180 and satisfies both conditions.

Alternate Solution:

Since the remainder when n is divided by 13 is 2, n = 13Q + 2 for some Q. Similarly, since the remainder when n is divided by 8 is 5, n = 8R + 5 for some R. Then, n + 11 = 13Q + 13 = 8R + 16 is divisible by both 8 and 13. Thus, n + 11 is divisible by the LCM of 8 and 13, which is 104. If n + 11 = 104, then n = 93. If n + 11 = 208, then n = 197, which is already greater than 180. Thus, there is only one possible value of n under 180.

Answer: B

Scott Woodbury-Stewart
Founder and CEO
[email protected]

Image

See why Target Test Prep is rated 5 out of 5 stars on BEAT the GMAT. Read our reviews

ImageImage